4. Breast Recon - 2010

¡Supera tus tareas y exámenes ahora con Quizwiz!

From 2013 pic is woman with radiated left chest wall A 74-year-old woman comes to the office because of a 2-month history of increasing soreness over the left lateral chest wall. History includes bilateral radical mastectomy and adjuvant radiation to the left chest 30 years ago because of left-sided breast cancer. On examination, a small, nonhealing 1 × 2-cm wound is visible in the subaxillary region within an area of dense fibrotic skin. A photograph is shown. Which of the following is the most appropriate next step? A) Biopsy of the nonhealing wound B) Coverage with a left latissimus muscle flap and skin grafting C) LHyperbaric oxygen treatment D) MRI of the chest wall E) Oral administration of antibiotics for 7 days

The correct response is Option A This patient's presentation is consistent with late radiation tissue injury (LRTI), which occurs in 5 to 15% of long-term cancer survivors who have received radiation and can occur months to years (even decades) after treatment. It can vary significantly with age, dose, and site of treatment. Characteristics include progressive tissue deterioration secondary to decreased vascularity followed by replacement of normal soft-tissue architecture by dense fibrotic tissue until there is insufficient oxygen delivery to sustain normal function. When LTRI of the chest wall progresses to skin breakdown and ulceration, as has happened in this patient, a biopsy is always required to rule out recurrence of the primary tumor or a radiation-induced squamous cell carcinoma or soft-tissue sarcoma. Hyperbaric oxygen therapy has been proposed as a treatment modality that can improve tissue quality and prevent tissue breakdown in irradiated areas. While most data on this relate to treatment of osteoradionecrosis of the mandible, there have been some reports of its use on the chest wall after irradiation due to breast cancer. However, the question specifically asks for the most appropriate next step, which would be biopsy rather than hyperbaric oxygen therapy. The principles of management once malignancy has been ruled out include debridement of necrotic tissues (including ribs) and reconstruction with well-vascularized flaps. In this case, a left latissimus muscle flap and skin graft was used for reconstruction after aggressive debridement. No alloplastic material was placed or thoracic cage reconstruction performed. This is common in these types of patients as excessive fibrosis caused by the radiation to the chest wall prevents loss of respiratory efficiency through paradoxical motion. CT scan or MRI may be useful in evaluating the size, extent, and nature of the problem, but does not take priority over tissue biopsy. The redness that is visualized on the patient's skin in the case is related to post-radiation changes and telangiectasias, which are common in LTRI, and is not infection. Therefore, antibiotics would be unnecessary.

An otherwise healthy 16-year-old girl comes to the office because of a painless mass in the left breast. Physical examination of the left breast discloses a circumscribed firm, rubbery, 3-cm mass without overlying skin changes, and no axillary lymphadenopathy. Results of a pregnancy test are negative. Which of the following is the most likely diagnosis? A) Common fibroadenoma B) Giant fibroadenoma C) Lactating adenoma D) Phyllodes tumor E) Tubular adenoma

The correct response is Option A. A common fibroadenoma is the most likely diagnosis of this patient. Common fibroadenoma is the most common breast tumor in adolescent females and present between the ages of 14 and 16. Juvenile fibroadenoma is a variant of fibroadenoma and is usually seen in adolescents and young adults. It is associated with a normal stromal/epithelial balance, which distinguishes it from phyllodes tumor, and has both stromal and epithelial hyperplasia. In addition, juvenile fibroadenomas are characterized by rapid growth. A giant fibroadenoma is a clinical diagnosis, rather than a pathologic diagnosis. It is characterized by its size, usually greater than 5 cm. Complex fibroadenoma is characterized by fibrocystic changes on glandular tissue with underlying features of common fibroadenoma on pathologic analysis. A tubular adenoma has glandular proliferation on pathologic analysis, and while it is a subtype of fibroadenoma, it is not as common as common fibroadenoma. A lactating adenoma, similarly, is defined by the presence of secretory hyperplasia of lobules on pathologic analysis. Lactating adenomas are so defined because of the histologic presence of secretory hyperplasia, and they commonly occur in pregnant or lactating women. Many lactating adenomas will spontaneously regress. A phyllodes tumor is typically a large, rapidly growing lesion and can be either benign or malignant. It is rare in adolescents, but when found, is usually aggressive. Phyllodes tumor is related to fibroadenoma and is distinct from other forms of breast cancer. Treatment is wide local excision. References Cerrato F, Labow, BI. Diagnosis and Management of Fibroadenomas in the Adolescent Breast. Semin Plast Surg. 2013 Feb; 27(1):23-5. Chang DS, McGrath MH. Management of benign tumors of the adolescent breast. Plast Reconstr Surg. 2007 Jul;120(1):13e-19e. Shermak MA, ed. Chapter 2: Congenital and Developmental Abnormalities of the Breast. In: Jatoi I, Kaufmann M, eds. Management of Breast Diseases. Springer-Verlag. 2010.

A 57-year-old woman with a history of modified left radical mastectomy followed by adjuvant chemotherapy and radiation therapy is evaluated six months after her last radiation treatment for breast reconstruction. Physical examination shows slight skin redundancy of the chest wall and grade 2 ptosis of the contralateral breast. She wears a size 36D brassiere and does not want to undergo symmetry procedures on the contralateral breast. Which of the following reconstructive options is most likely to achieve breast symmetry in this patient? A) Deep inferior epigastric perforator flap B) Latissimus dorsi flap C) Thoracodorsal artery perforator flap D) Tissue expander with acellular dermal matrix and staged implant placement E) Total submuscular tissue expander and staged implant placement

The correct response is Option A. Breast reconstruction requires reconstruction of both the soft-tissue envelope and volume. In the setting of radiated chest wall tissues, it would be difficult to recreate a breast envelope of sufficient size to match the contralateral breast with tissue expansion, even with acellular dermal matrix or total submuscular placement of a tissue expander. The history of radiation would likely cause an element of capsular contracture and decrease symmetry to the contralateral breast with grade 2 ptosis. Both the latissimus dorsi flap and the thoracodorsal artery perforator flap would recruit non-radiated skin to create an adequate envelope, but would not provide adequate volume alone. The deep inferior epigastric perforator (DIEP) flap would provide both envelope and volume.

When performing immediate breast reconstruction, it is important to reconstruct the lateral inframammary fold. This is because the oncologic extirpation of the breast is carried out to which of the following anatomic landmarks? A) Anterior edge of the latissimus dorsi muscle B) Anterior edge of the serratus muscle C) Lateral edge of the pectoralis major muscle D) Lateral edge of the pectoralis minor muscle E) Posterior edge of the serratus muscle

The correct response is Option A. For modified radical and simple mastectomies, the landmarks of dissection are: superiorly to the clavicle, medially to the sternum, inferiorly to the inframammary fold, and laterally to the border of the latissimus dorsi muscle. The pectoralis major muscle fascia is resected with the specimen. The recreation of the inframammary fold is important for shaping in breast reconstruction and care must be taken to evaluate and repair both the inferior and lateral components of the inframammary fold.

A 45-year-old woman with a T2 N0 M0 invasive ductal carcinoma in the inferior pole of the left breast is scheduled to undergo segmental mastectomy followed by radiation therapy. She currently wears a size 36E brassiere and wants to have any cup size from a C to D. After segmental resection of the tumor, which of the following procedures is most likely to yield the best cosmetic result in this patient? A) Bilateral reduction mammaplasty B) Primary closure of the left breast and reduction mammaplasty of the right breast C) Reconstruction of the left breast with a latissimus dorsi musculocutaneous flap and reduction mammaplasty of the right breast D) Reconstruction of the left breast with a transverse rectus abdominis musculocutaneous flap and mastopexy of the right breast E) A single-stage breast reconstruction with placement of a silicone prosthesis in the left breast and reduction mammaplasty of the right breast

The correct response is Option A. In a patient with large breasts, in whom a partial mastectomy is required, reduction mammaplasty is an appropriate management. This procedure will potentially relieve symptoms of macromastia, reduce the amount of breast tissue present in both breasts, and offer the best aesthetic outcome. Implantation of a prosthesis in a breast that will undergo radiation therapy increases the risk for complications and would likely lead to a less symmetrical result. The latissimus dorsi or transverse rectus abdominis musculocutaneous (TRAM) flaps could be used for immediate partial reconstruction, but they are ideal for delayed reconstruction of partial mastectomy defects. Reduction mammaplasty does not preclude future reconstruction options, but a latissimus flap reconstruction would. With a T2 tumor, a significant portion of the lower pole of the breast is removed to obtain clear margins. Even though the patient described has moderate-to-large breasts, there is a high likelihood that she will develop a deformity of the left breast and asymmetry with the right breast if no reconstruction is performed.

A 53-year-old woman comes to discuss breast reconstruction after undergoing left mastectomy for ductal carcinoma in situ. She does not require chemotherapy or radiation therapy and does not want surgery on the unaffected breast. She is obese but otherwise healthy with a large, ptotic right breast. Which of the following breast reconstruction techniques is most likely to result in the greatest long-term patient satisfaction? A) Autologous breast reconstruction with microvascular free tissue transfer from the abdomen B) Extended latissimus dorsi musculocutaneous flap reconstruction without an implant C) Immediate reconstruction with cohesive, anatomically shaped silicone gel implant D) Immediate tissue expander followed by implant-based reconstruction E) Latissimus dorsi musculocutaneous flap reconstruction with immediate placement of a permanent implant

The correct response is Option A. Multiple studies have supported improved patient satisfaction with autologous breast reconstruction in the setting of a unilateral reconstruction. The clinical scenario involves an obese patient with a large, ptotic breast who does not desire a surgical procedure for symmetry and is the ideal candidate for an autologous reconstruction. Microvascular free tissue transfer of abdominal tissue for breast reconstruction demonstrates improved reliability and decreased fat necrosis compared with pedicled flap reconstruction. Implant-based reconstruction is less likely to provide adequate symmetry in this patient. An extended latissimus dorsi musculocutaneous flap-based reconstruction alone is unlikely to provide enough volume for symmetry and a traditional latissimus flap with implant is unlikely to provide adequate ptosis for symmetry.

A 62-year-old woman undergoes breast reconstruction using autologous tissue from the abdomen. Intraoperatively, use of a perforator flap is found to be impossible because of multiple small nondominant perforators. Conversion to a delayed pedicled transverse rectus abdominis muscle flap is planned. Ligating which of the following vessels in this stage will best facilitate future viability of the tissue transferred in the next stage? A) Deep inferior epigastric B) Hypogastric C) Internal mammary D) Superficial inferior epigastric E) Superior epigastric

The correct response is Option A. Pedicled transverse rectus abdominis muscle (TRAM) flaps are based on the superior epigastric system, which is often less robust than the deep inferior epigastric system. Therefore, surgical delay by ligation of the deep inferior epigastric system may facilitate overall viability of the transferred tissue. Ligation of the superior epigastric system would make a pedicled TRAM flap unlikely to survive. The internal mammary ligation may also interrupt blood supply to the superior epigastric system, and even if the tissue is fed through collaterals, it would not strengthen the flap. Division of the superficial inferior epigastric system might also help, but it is not as critical as ligation of the deep inferior epigastric system. The hypogastric system does not have a direct impact on the pedicled TRAM tissues.

A 33-year-old woman comes to the office with a 6-cm rapidly growing tumor of the left breast. She wears a size 36C brassiere. The tumor has a bluish hue and skin veins are dilated. A phyllodes tumor is diagnosed, and surgical excision is planned. Which of the following is the most appropriate surgical procedure to treat this patient? A) Excision with 1-cm margin B) Excision with 2-cm margin C) Excisional biopsy D) Modified radical mastectomy E) Radical mastectomy

The correct response is Option A. Phyllodes are large benign tumors that occur primarily in the perimenopausal patient. Previously, they were referred to as cystosarcoma phyllodes, a term coined in 1838 because the tumors are fleshy and have a gross leaf-like intracanalicular growth pattern. However, this is a misnomer because these tumors do not behave like sarcomas and are rarely malignant. The histologic characteristics that separate fibroadenomas from phyllodes tumors are not well defined and have been somewhat controversial. Nevertheless, phyllodes tumors typically are large fibroadenomas that histologically have more stromal cellularity than that seen in the typical fibroadenoma. The classification of benign versus malignant phyllodes tumors is not sharply delineated, and the term borderline lesion may be more appropriate. Borderline lesions have more mitoses per high-power field and moderate nuclear pleomorphism. They have a tendency to recur after local excision but do not demonstrate true malignant behavior. When metastases of a phyllodes tumor have been reported, there have been obvious sarcomatous elements such as liposarcoma or rhabdomyosarcoma in the lesion. The surgical treatment of phyllodes tumors has recently been redefined. In the past, simple or radical mastectomies were recommended for the treatment of large phyllodes tumors. Currently, most surgeons perform more conservative surgery. Several clinical studies have recommended the excision of tumors with 1-cm clear margins or mastectomy if breast conservation is impossible.

A 45-year-old man comes to the office for consultation regarding breast cancer after undergoing gene testing and learning that he is a carrier of the BRCA2 gene mutation. He has a strong family history of breast, prostate, and ovarian cancers. Which of the following best represents his lifetime risk for developing breast cancer? A ) 6% B ) 15% C ) 30% D ) 60% E ) 85%

The correct response is Option A. Since the identification of the first breast cancer gene in 1990, the field of molecular breast cancer testing has grown enormously. It is estimated that approximately 5% to 10% of patients diagnosed with breast cancer last year carry highly penetrant cancer susceptibility genes. It is important to identify these patients early, as the presence of genetic mutations or other high-risk factors may alter management strategies for patients, both prior and subsequent to the diagnosis of breast cancer. Male breast cancer accounts for less than 1% of all breast cancer cases in the United States and is more common in men with a family history of breast cancer. The relationship between male breast cancer and a deleterious BRCA2 mutation has been well established. In the largest study to date, the lifetime risk of developing breast cancer in a BRCA2 male carrier is approximately 7%. The correlation with BRCA1 and male breast cancer is less clear, but it seems that the presence of BRCA1 mutation confers a lower lifetime risk of developing breast cancer than a BRCA2 mutation. The relative risk of developing breast cancer is highest for men in their 30s and 40s, and it decreases with increasing age. The lifetime risk for the development of breast cancer in female carriers of BRCA1 and BRCA2 mutations is significantly higher than for males. Female BRCA1 carriers have an 85% risk of developing breast cancer and a 62% risk of developing ovarian cancer. Female BRCA2 carriers have an 85% risk of developing breast cancer and a 25% risk of developing ovarian cancer. References

A 39-year-old woman with a history of fibrocystic breast lesions comes to the office for consultation. She has no family history of breast cancer. Results of routine mammograms have been negative; she has never undergone biopsy. Which of the following is the most appropriate recommendation for this patient regarding managing her risk of breast cancer? A) Continue to schedule routine mammograms B) Refer for mastectomy C) Schedule core needle biopsy D) Schedule fine-needle aspiration E) Schedule genetic testing

The correct response is Option A. Studies have shown the fibrocystic changes alone in the breast are not directly linked to an increased risk of breast cancer, so there is no indication for fine-needle aspiration or core biopsy. Cancer risk increases in benign breast disease with increased proliferation and atypical hyperplasia. Even with no family history of breast cancer, it is recommended that the patient continue routine mammograms. Fibrocystic breast disease has not been linked in the literature to an increased risk of mutations of the BRCA genes; therefore, genetic testing is unnecessary.

A 45-year-old woman undergoes mastectomy. Which of the following anatomical landmarks denotes a limit of the breast and, therefore, a limit to the extent of the mastectomy? A) Anterior border of latissimus dorsi muscle B) Inferior origin of pectoralis major muscle C) Lateral pectoralis minor muscle D) Superficial fascia of the serratus anterior muscle E) Supraclavicular lymph node basin

The correct response is Option A. The borders of the breast and, therefore, the limits of the mastectomy are the sternum, clavicle, inframammary fold, and anterior border of the latissimus dorsi muscle. The other options are not designated as borders of the breast tissue.

A 52-year-old woman is evaluated for breast reconstruction after modified radical mastectomy with adjuvant chemotherapy and radiation therapy 18 months ago. Her last radiation treatment was 8 months ago. BMI is 29 kg/m2. Examination today shows hyperpigmentation of the right chest wall with no redundancy of the mastectomy skin flaps. Her contralateral breast is a D cup with grade III ptosis. Which of the following methods will create the best symmetry for this patient? A) Deep inferior epigastric perforator flap B) Gel breast implant and acellular dermal matrix C) Latissimus dorsi musculocutaneous flap D) Tissue expander and acellular dermal matrix placement with planned staged exchange for permanent gel implant E) Tissue expander placement alone with planned staged exchange for permanent gel implant

The correct response is Option A. The deep inferior epigastric perforator flap would give the patient autologous tissue reconstruction with ample tissue for skin resurfacing and soft tissue for volume. In this radiated patient with a tight skin envelope, a tissue expander/implant, with or without acellular dermal matrix, would be difficult to create an appropriately ample skin envelope and the patient would be at higher risk for wound-healing problems and capsular contracture. The latissimus dorsi musculocutaneous flap, although an autologous tissue reconstruction, would have insufficient volume to adequately match this patient's contralateral side. It would have to be combined with an implant.

An otherwise healthy 38-year-old woman undergoes prophylactic bilateral mastectomy and immediate reconstruction with deep inferior epigastric artery perforator (DIEP) free flaps. Intraoperatively, the left DIEP flap appears congested before the conclusion of the case. The left deep inferior epigastric artery and vein (DIEA and DIEV) were anastomosed to the proximal internal mammary vessels. The vascular pedicle is evaluated and each anastomosis appears patent and not kinked; however, the venous congestion persists. Which of the following is the most appropriate management? A) Anastomose the superficial inferior epigastric vein to an internal mammary vessel perforator B) Convert to left prosthetic reconstruction C) Infuse tissue plasminogen activator (tPA) to the DIEA D) Initiate leech therapy E) Revise the DIEV anastomosis to the retrograde internal mammary vessel limb

The correct response is Option A. Venous drainage of the lower abdominal skin and subcutaneous tissue occurs primarily through the superficial venous system and secondarily through the deep venous system, with perforating veins interconnecting the two systems. These communicating veins have been identified on computed tomography angiography in approximately 90% of abdominal walls in vivo. The majority of the remaining 10% of patients likely have communicating veins that are too small to visualize or are absent. In these cases of anatomical superficial venous system dominance, venous drainage is dependent on the superficial venous system. A recently published 2012 article by Sbitany et al. demonstrated that the incidence of intraoperative venous congestion secondary to persistent superficial venous system dominance was 0.9% in 1201 muscle-sparing transverse rectus abdominis musculocutaneous and deep inferior epigastric artery perforator free flaps. A free flap that becomes congested after reperfusion in the operating room should be assessed immediately for possible etiologies including twisting, kinking, tension, or vasospasm of the vascular pedicle. If a technical problem is ruled out and the venous anastomosis remains patent, obligatory enhancement of venous drainage with the superficial venous system is necessary to salvage the free flap rather than revision of the original anastamosis. Various methods include an anastomosis of the superficial inferior epigastric vein (SIEV) to the DIEV system or any chest wall vein, including the retrograde limb of the internal mammary vessel, the branch of the internal mammary vessel, or the thoracodorsal system. This requires preemptive planning and sparing of the superficial epigastric vein or SIEV during the dissection of the flap. A vein graft can be utilized if additional length is necessary. Another option is to substitute the DIEV anastomosis with the SIEV. Tissue plasminogen activator (tPA) would not be indicated in this scenario, as it is used as a thrombolytic and there is no evidence of vascular thrombosis. Revising the DIEV anastomosis would be moot because it is patent and the deep system is being drained. Leech therapy is useful for venous congestion, but primarily as an adjunct after potential surgical etiologies have been addressed. Sacrificing the free flap without first attempting salvage is not warranted, and using a prosthetic would be possible only if prior patient consent were obtained.

A 26-year-old woman with a strong family history of breast cancer undergoes genetic testing. She is found to have a deleterious mutation of the BRCA1 gene. Which of the following best describes her lifetime risk for the development of breast cancer when compared with women without this mutation? A) Two times greater B) Six times greater C) Ten times greater D) Fifteen times greater E) Twenty times greater

The correct response is Option B. A woman's lifetime risk of developing breast and/or ovarian cancer is greatly increased if she inherits an altered BRCA1 or BRCA2 gene. Women with an inherited alteration in one of these genes have an increased risk of developing these cancers at a young age (before menopause) and often have multiple close family members with the disease. These women may also have an increased chance of developing colon cancer. Men with an altered BRCA1 or BRCA2 gene also have an increased risk of breast cancer (primarily if the alteration is in BRCA2) and possibly prostate cancer. Alterations in the BRCA2 gene have also been associated with an increased risk of lymphoma, melanoma, and cancers of the pancreas, gallbladder, bile duct, and stomach in some men and women. According to estimates of lifetime risk, approximately 13.2% (132 of 1000 individuals) of women in the general population will develop breast cancer, compared with estimates of 36 to 85% (360 to 850 of 1000) of women with an altered BRCA1 or BRCA2 gene. In other words, women with an altered BRCA1 or BRCA2 gene are 3 to 7 times more likely to develop breast cancer than women without alterations in those genes. Lifetime risk estimates of ovarian cancer for women in the general population indicate that 1.7% (17 of 1000) will get ovarian cancer, compared with 16 to 60% (160 to 600 of 1000) of women with altered BRCA1 or BRCA2 genes. No data are available from long-term studies of the general population comparing the cancer risk in women who have a BRCA1 or BRCA2 alteration with women who do not have an alteration in these genes. Therefore, these figures are estimated ranges that may change as more research data are added.

An otherwise healthy 38-year-old woman who is a smoker is considering implant-based breast reconstruction following mastectomy. She has been counseled about likely use of acellular dermal matrix when the intermediate tissue expander is placed and wants to further understand why the matrix will be used in her body. The patient should be advised that use of acellular dermal matrix is associated with a decreased risk for which of the following? A) Cancer recurrence B) Capsular contracture C) Seroma D) Skin flap necrosis E) Smoking-related complications

The correct response is Option B. Acellular dermal matrix (ADM) use has been increasing over time with tissue expander or implant-based breast reconstruction following mastectomies. Many potential advantages and disadvantages have been studied, and some of the data are contradictory. However, the consensus of the literature indicates that ADMs are associated with decreased capsular contracture rates. There is literature to suggest that seroma rates are increased or remain stable, not decreased, with ADMs. ADMs have not been shown to decrease independent patient risk factors for complications such as tobacco use or to decrease cancer recurrence rates. ADMs also do not appear to improve vascularity of the tissue overlying them when initially placed.

A 42-year-old woman is evaluated because of an invasive cancer of the right breast. Which of the following best approximates the likelihood that this patient's cancer is associated with the BRCA1 or BRCA2 genes? A) 1% B) 10% C) 25% D) 40% E) 75%

The correct response is Option B. Among familial breast cancers, 5 to 10% are considered to be hereditary. These familial breast cancers are linked to specific mutations on a cancer susceptibility gene. The breast cancer susceptibility genes (BRCA) belong to a class of genes known as tumor suppressors. In normal cells, BRCA1 and BRCA2 genes stabilize the DNA and prevent uncontrolled cell growth. A woman's lifetime risk of developing breast and/or ovarian cancer is greatly increased if she inherits a mutation on BRCA1 or BRCA2 genes. BRCA1- and BRCA2-related breast cancers occur in younger women and are often associated with estrogen receptor-negative tumors.

A 35-year-old woman comes to the office to discuss a recent diagnosis of breast cancer. Recent mammography showed diffuse microcalcifications throughout the breast, and needle biopsy showed infiltrating ductal carcinoma. On physical examination, some retraction of the skin in the lower outer quadrant of the breast is noted. She wears a size 36C brassiere. The patient reports that she is currently considering whether to have lumpectomy and radiation therapy or mastectomy. Which of the following features of this clinical scenario is a CONTRAINDICATION to breast conservation therapy? A) Breast size B) Mammographic findings C) Patient age D) Skin retraction on physical examination of the breast E) Tumor pathology

The correct response is Option B. Breast conservation therapy (BCT) refers to breast conserving therapy followed by moderate-dose radiation to eradicate microscopic residual disease. The goal is to provide the equivalent survival of mastectomy while maintaining a cosmetically acceptable appearance with a low rate of recurrence. When considering breast conservation therapy or mastectomy, the needs and desires of each patient should be addressed. Age alone is not a contraindication to BCT, but overall health and comorbidities should be considered. Histologic subtype and pathology are not contraindications to BCT as long as the tumor is not diffuse and can be safely excised with negative margins. Similarly, breast size needs to be considered along with tumor size and location, but "small" or "large" breasts are not indications or contraindications. While retraction of the skin, nipple, or breast parenchyma is not an absolute contraindication to BCT, as long as negative margins can be safely removed, the cosmetic impact of their involvement should be considered. There are few absolute contraindications to BCT, but they include: Multicentric disease with two or more tumors in separate quadrants of the breast such that they cannot be encompassed in a single excision Diffuse malignant microcalcifications on mammography A history of prior radiation in the same breast or chest wall Pregnancy Persistently positive margins despite re-excision References Nijenhuis MV1, Rutgers EJ. Who should not undergo breast conservation? Breast. 2013 Aug;22 Suppl 2:S110-4. S. Litière, G. Werutsky, I.S. Fentiman, E. Rutgers, M.-R. Christiaens, E. Van Limbergen et al. Breast conserving therapy versus mastectomy for stage I-II breast cancer: 20 year follow-up of the EORTC 10801 phase 3 randomised trial. Lancet Oncol, 2014 Apr: 412-419.

A 52-year-old woman receives a diagnosis of invasive ductal carcinoma of the right breast. Which of the following details from this patient's history is the strongest risk factor for this diagnosis? A) Early first pregnancy (less than 30 years) B) Early menarche (less than 12 years) C) Early menopause (less than 55 years) D) Multiple episodes of breast-feeding E) Remote oral contraceptive use

The correct response is Option B. Early menarche is the highest risk factor for breast cancer of the options listed. Late first pregnancy, late menopause, no breast-feeding, and recent oral contraceptive use are also risk factors for breast cancer but are not as high risk.

PIc of girl with huge left breast A 12-year-old girl with Beckwith-Wiedemann syndrome develops profound breast enlargement at the onset of puberty. Physical examination shows two distinct masses in the right breast and severe hyperplasia consuming the left breast. A photograph is shown. Which of the following is the most appropriate surgical management? A) Hormone suppression therapy B) Right lumpectomies; left mastectomy with skin reduction with application of nipple as skin graft C) Right simple mastectomy; left mastectomy with skin reduction with application of nipple as skin graft D) Right simple mastectomy; left simple mastectomy with sentinel lymph node biopsy E) Right total mastectomy; left modified radical mastectomy

The correct response is Option B. Epithelial hyperplasia is a benign pathological process. Therefore, modified radical mastectomy or sentinel lymph node biopsy would not be indicated in the absence of cancer. Mastectomy on the right is not indicated because the masses are discrete and separate from the normal breast parenchyma. As such, a viable breast mound can be preserved in this 12-year-old girl. Because of the expansive nature of her hyperplasia on the left, skin reduction is required to ensure smooth contour of her skin flaps. The nipple may be spared because of the benign disease. Reconstruction can be performed in a delayed fashion after development is complete. An adjustable expander/prosthesis can also provide a reasonable breast mound until definitive reconstruction is performed. Hormone suppression therapy may temporarily slow the growth of the hyperplasia but could adversely affect this child's normal development. It would not comprise definitive management.

A 35-year-old BRCA-positive woman undergoes bilateral prophylactic nipple-sparing mastectomy via lateral radial incisions with periareolar extensions. She undergoes direct-to-implant reconstruction with an acellular dermal matrix inferior lateral sling. Mastectomy specimens weigh 435 g each, and placement of 450 mL smooth, round, high-profile implants is performed. Postoperatively, 25% partial nipple areolar complex necrosis in both breasts is noted. Which of the following most likely contributed to the necrosis? A) Bilateral surgery B) Mastectomy incision C) Use of acellular dermal matrix D) Volume of the implants E) Weight of the mastectomy specimen

The correct response is Option B. Nipple-sparing mastectomies are becoming more prevalent with the rise of prophylactic mastectomies and increased comfort with performing it in presence of in situ and invasive carcinoma. While universal standards for patient selection do not exist, there is consensus that general recommendations include no inflammatory breast cancers, no pathologic nipple discharge, no Paget's disease, and for many, a 2-cm distance from the tumor to the nipple. Nipple-sparing mastectomy incisions have been described as radial lateral, lateral inframammary fold, and peri-areolar. Because the entire skin envelope is preserved, direct-to-implant reconstruction is possible with inferolateral support from acellular dermal matrices or other scaffolds. Areolar involvement in mastectomy incisions is associated with increased rates of nipple necrosis. The weight of mastectomy specimen, volume of implants, use of acellular dermal matrices or laterality of surgery have not been shown to be associated with nipple necrosis.

A 54-year-old woman with BRCA mutation is scheduled for a bilateral nipple-sparing mastectomy. Which of the following would lead to the highest risk for postoperative nipple necrosis? A) Grade 2 breast ptosis B) Periareolar mastectomy incision C) Previous excisional breast scar D) Tumor greater than 1.5 cm E) Use of smooth round gel implants

The correct response is Option B. Nipple-sparing mastectomy and direct to implant reconstruction is becoming increasingly popular in the setting of prophylactic mastectomies. The criteria for nipple-sparing mastectomies have been increased to tumors not involving the nipple-areola complex, with some surgeons requiring a distance of 2 cm from the nipple and others espousing just a clean surgical margin at the nipple. Nipple-sparing mastectomies do not require recreating the breast envelope and therefore direct to implant reconstruction is possible. Increased risk for incidence of nipple necrosis in the setting of direct to implant reconstruction is associated with mastectomy incision involving the areola. The use of smooth round gel implants, grade 2 ptosis, tumor size, or previous breast biopsy scars are not associated with nipple necrosis. References Colwell AS, Damjanovic B, Zahedi B, et al. Retrospective review of 331 consecutive immediate single-stage implant reconstructions with acellular dermal matrix: indications, complications, trends, and costs. Plast Reconstr Surg. 2011 Dec;128(6):1170-8. Colwell AS, Tessler O, Lin AM, et al. Breast reconstruction following nipple-sparing mastectomy: predictors of complications, reconstruction outcomes, and 5-year trends. Plast Reconstr Surg. 2014 Mar;133(3):496-506.

Oral contraceptive use increases the incidence of which of the following histologic risk factors for breast cancer? A) Apocrine metaplasia B) Atypical hyperplasia C) Intraductal hyperplasia D) Intraductal papilloma E) Sclerosing adenosis

The correct response is Option B. Patients in whom an otherwise benign breast biopsy shows atypical hyperplasia have a 4.5- to 5-fold increased risk for developing breast cancer. Proliferative breast disease without atypia increases the risk 1.5- to 2-fold. Oral contraceptives have shown to decrease the occurrence of all proliferative forms of benign breast disease without atypia, such as intraductal hyperplasia, intraductal papilloma, and sclerosing adenosis. Apocrine metaplasia is a non-proliferative histologic change and carries no increased risk for breast cancer. Of the choices, only atypical hyperplasia is increased with oral contraceptive use. References Rohan TE, Miller AB. A cohort study of oral contraceptive use and risk of benign breast disease. Int J Cancer. 1999 Jul 19;82(2):191-196. Schnitt SJ. Benign breast disease and breast cancer risk: potential role for antiestrogens. Clin Cancer Res. 2001 Dec;7(12 Suppl):4419s-4422s. Vamre TB, Stalsberg H, Thomas DB, et al. Extra-tumoral breast tissue in breast cancer patients: variations with steroid contraceptive use. Int J Cancer. 2006 Jun 1;118(11):2827-2831.

A 52-year-old woman with breast cancer undergoes right mastectomy and reconstruction with a free transverse rectus abdominis musculocutaneous (TRAM) flap. The procedure is uneventful. In the recovery room, the patient's husband says that she has been smoking one pack of cigarettes daily up to the day of surgery. Which of the following postoperative complications is most likely to occur? A) Flap hematoma B) Mastectomy skin loss C) Microvascular thrombosis D) Partial flap loss E) Superficial infection

The correct response is Option B. Patients who smoke cigarettes and who undergo breast reconstruction with a free flap have a higher rate of mastectomy skin loss. In addition, they have a high rate of donor site abdominal flap necrosis, umbilical necrosis, and hernias. There is no increase in microsurgical complications, flap-related complications (partial flap loss or fat necrosis), infections, or hematomas. Current recommendations are for patients to stop smoking at least 4 weeks prior to breast reconstruction. Patients who quit smoking prior to this period have a lower risk of perioperative complications when compared to active smokers.

A 53-year-old woman is referred for consultation regarding breast reconstruction following mastectomy. Autologous breast reconstruction options are discussed. Which of the following is an advantage of the pedicled transverse rectus abdominis musculocutaneous (TRAM) flap over the free TRAM flap? A) Decreased complications in patients with a history of smoking B) Decreased incidence of complete flap loss C) Decreased risk of abdominal hernia D) Decreased risk of fat necrosis E) Ease of insetting and flap shaping

The correct response is Option B. There is considerable controversy regarding the merits of the pedicled TRAM, free TRAM, free deep inferior epigastric perforator (DIEP), and free superficial inferior epigastric perforator (SIEP) flap reconstructions. Reconstruction with each flap has distinct advantages and disadvantages. Currently, the literature does not clearly favor the use of one procedure over the others. The pedicled TRAM flap is characterized by a shorter operative time and a decreased risk of complete flap loss when compared to the free flaps. However, the pedicled TRAM flap shows a higher incidence of fat necrosis with partial flap loss, as well as an increased length of stay. Free flaps offer ease of flap shaping and insetting, as well as a decreased risk of abdominal hernia and abdominal wall weakness. Free flaps are more suitable for patients who are diabetic, are overweight, or smoke cigarettes. Deciding which particular procedure to perform requires assessment on a case-by-case basis and consideration of the surgeon's level of comfort with the different techniques.

A 42-year-old woman comes to the office for treatment after receiving a diagnosis of cancer of the right breast. She has decided to undergo mastectomy of the right breast. Which of the following is a relative CONTRAINDICATION to nipple-sparing mastectomy? A) Comedo-type breast tumor B) Invasive lobular carcinoma C) Subareolar tumor D) Tumor location 3 cm from the nipple E) Tumor size of 2.5 cm

The correct response is Option C. A relative contraindication to nipple-sparing mastectomy is a centrally located tumor. Although various authors have employed different distance criteria, it is generally accepted that patients whose tumors are within 2 cm of the nipple are not candidates for nipple-sparing mastectomy. Nipple-sparing mastectomy is an appropriate option for high-risk patients undergoing prophylactic mastectomy and for patients diagnosed with breast cancer who meet certain criteria. Those criteria are: tumor size of 3 cm or less, at least 2 cm from the nipple, not multicentric, and with clinically negative nodes. Comedo carcinoma of the breast is a type of ductal carcinoma in situ. It is considered to be an early stage of breast cancer, is confined to the ducts, and usually does not spread beyond. It is not a contraindication to nipple-sparing mastectomy. Invasive lobular carcinoma originates from the breast lobules, may form a thickening of the breast tissue rather than a discrete mass, and is often bilateral. As long as it meets the above criteria, it is not a contraindication to nipple-sparing mastectomy. Inflammatory breast cancer, Paget disease, and tumors infiltrating the skin are also not candidates for skin-sparing or nipple-sparing mastectomy, according to several authors. In more recent studies, a tumor size of 3 cm or less appears to result in no increase in local or regional recurrence in nipple-sparing mastectomy compared with alternative surgical approaches. A tumor of 2.5 cm is not a contraindication to nipple-sparing mastectomy.

A 42-year-old woman comes to the office because she is dissatisfied with an obvious step-off between the chest wall and the superior pole of the breast 6 months after she underwent immediate expander reconstruction of the left breast. No further adjuvant therapy was indicated. After full expansion, the tissue expander was removed, and a permanent smooth, round silicone prosthesis was placed. BMI today is 28 kg/m2. Examination shows a well-defined inframammary fold. The volume is matched to the contralateral breast. Which of the following is the most appropriate surgical procedure for correction of this patient's deformity? A) Exchange of the silicone prosthesis with a silicone prosthesis of a larger volume and dimension B) Exchange of the silicone prosthesis with a saline prosthesis of the same volume and dimension C) Fat grafting to the superior pole and chest wall D) Placement of acellular dermal matrix as an inferolateral sling E) Removal of the prosthesis and reconstruction with a deep inferior epigastric perforator flap

The correct response is Option C. Fat grafting has been shown to successfully address acquired contour deformities in breast reconstruction. Changing the prosthesis to a saline prosthesis of the same size and dimension will not address the problem of the step-off between the prosthesis and the chest wall in this slender patient (BMI 20 kg/m2) which is due to decreased soft-tissue coverage in the superior pole. Likewise, increasing the size and dimensions of the silicone prosthesis would not correct this contour deformity and would also lose the symmetry with the contralateral breast. This patient, without excess lower abdominal skin and subcutaneous fat, is not a candidate for a deep inferior epigastric perforator flap. Placement of acellular dermal matrix as an inferolateral sling is typically performed in the initial stage of tissue expansion and reconstruction. It can help define the inframammary fold, which this patient does not need.

An otherwise healthy 52-year-old woman is scheduled to undergo bilateral breast reconstruction with abdominal perforator flaps. She does not smoke cigarettes. Which of the following is the most appropriate antibiotic prophylaxis regimen? A) Preoperative antibiotics B) Preoperative chlorhexidine bath C) Preoperative and intraoperative antibiotics D) Preoperative, intraoperative, and postoperative antibiotics for 7 days E) Preoperative, intraoperative, and postoperative antibiotics until all drains are removed

The correct response is Option C. For a healthy patient undergoing a lengthy autologous breast reconstruction, the most appropriate regimen of antibiotic prophylaxis consists of a preoperative dose of intravenous antibiotic and intraoperative antibiotics to ?maintain therapeutic levels during operation and, at most, a few hours after closure? (Mangram, et al). Although preoperative chlorhexidine bathing is supported by some studies and strong theoretical rationale, it is not nearly as effective as antibiotic prophylaxis on its own. A single dose of preoperative antibiotic intravenously is appropriate for short procedures; however, therapeutic levels of antibiotic should be maintained throughout the lengthy surgery described. Continued administration of prophylactic antibiotics for 7 days following surgery or until drains are removed has not been proven to reduce the incidence of surgical site infections. This regimen also promotes drug resistance and adverse patient reactions.

A 36-year-old woman with genetic susceptibility to breast cancer is scheduled to undergo bilateral prophylactic mastectomy. She has elected to proceed with immediate single-stage reconstruction using permanent silicone implants and acellular dermal matrix. Which of the following is the most sensitive method to detect implant rupture in this patient? A) CT scan B) Mammography C) MRI D) Physical examination E) Ultrasonography

The correct response is Option C. Implant rupture is one of the most common reasons for implant removal. While implant rupture can be associated with symptoms such as capsular contracture, it is often completely asymptomatic or "silent." For this reason, screening imaging to detect such ruptures has been recommended. According to the 2011 Update on the Safety of Silicone Gel-Filled Breast Implants published by the U.S. Food and Drug Administration, it is currently recommended that women with silicone implants get their first MRI 3 years after they receive the implants and every 2 years thereafter to detect silent ruptures. Since the rate of rupture increases the longer an implant is in place, not screening is unacceptable. Mammograms can detect extracapsular silicone when an implant ruptures, but they do not detect intracapsular ruptures. The accuracy of ultrasound largely depends on the skill of the ultrasound technologist, the type of equipment used, and the experience of the interpreting physician. Furthermore, ultrasound is limited in its ability to detect ruptures in the back wall of the implant and in the breast tissue behind it. CT scans can detect intracapsular silicone gel-filled breast implant rupture, but they are limited in their ability to detect extracapsular ruptures.

A 41-year-old man is referred to the office because of a mass on his left breast that has been growing rapidly for 1 month. Examination of the specimen obtained on core needle biopsy confirms invasive mammary carcinoma. Genetic testing results are positive for BRCA2. In addition to an increased risk of male breast cancer, which of the following other types of cancer is most likely to be associated with this patient? A) Colon B) Lung C) Pancreatic D) Renal E) Thyroid

The correct response is Option C. Men with harmful BRCA1 mutations also have an increased risk of breast cancer and, possibly, pancreatic cancer, testicular cancer, and early onset prostate cancer. However, male breast cancer, pancreatic cancer, and prostate cancer appear to be more strongly associated with BRCA2 gene mutations. Colon, lung, renal, and thyroid cancers are not associated with the BRCA1 and BRCA2 gene mutations.

A 21-year-old woman comes to the office for consultation regarding a palpable lump in the left breast that she first noticed 4 months ago. Phyllodes tumor of the breast is diagnosed. Which of the following is the most appropriate management at this time? A ) Observation and incisional biopsy B ) Neoadjuvant chemotherapy C ) Local excision and annual surveillance D ) Wide local excision and radiation therapy E ) Mastectomy with immediate reconstruction

The correct response is Option C. Phyllodes tumors of the breast were once called cystosarcoma phyllodes because of the fleshy nature of the tumor. Because most tumors are benign, the name may be misleading. Thus, the favored terminology is now phyllodes tumor. Phyllodes tumor is the most commonly occurring nonepithelial neoplasm of the breast but represents only about 1% of tumors in the breast. It has a sharply demarcated, smooth texture and is typically freely movable. It is a relatively large tumor; the average size is 5 cm. However, lesions greater than 30 cm have been reported. Approximately 90% of phyllodes tumors are benign and approximately 10% are malignant. In considering excision, the tumor-to-breast ratio should be small enough to allow for segmental excision of the tumor and possible reduction mammaplasty technique (eg, inferior pedicle for a superior tumor or a superior pedicle for an inferior lesion). Annual surveillance and follow-up is recommended because the pathologic appearance does not always predict the clinical behavior.

2016 and below A 52-year-old woman undergoes immediate unilateral breast reconstruction with a free deep inferior epigastric flap. Near completion of the procedure, the flap skin paddle is noted to have venous congestion. On exploration of the pedicle, the anastomosis between the vena comitans and the internal mammary vessel appears patent. Which of the following preventive measures would most likely have averted this issue? A) Administration of heparin immediately before release of vessel clamps and flap revascularization B) Anastomosis of the veins using sutures instead of a venous coupling device C) Preservation and anastomosis of the superficial inferior epigastric vein D) Routine anastomosis of two venae comitantes per flap E) Use of near-infrared fluorescence imaging to assess flap blood flow

The correct response is Option C. Preservation of the superficial inferior epigastric veins (SIEV) during flap harvest is a useful preventive measure in microsurgical free tissue transfer operations. These veins can serve as important lifeboats to augment venous outflow in the setting of venous congestion. Typically, if a free flap demonstrates venous congestion, the inset should be taken down and the pedicle, recipient vessels, and anastomoses interrogated. Simple issues, such as mechanical compression or twisting of the vein should be ruled out. Next, the SIEV should be inspected. If it is engorged, it is likely that the flap is reliant on superficial outflow, and this vein should be connected to a recipient vessel to augment the venous outflow of the flap. Options for recipient veins include an anterograde branch on the pedicle vena comitans, or in a retrograde fashion to the vena comitans that was not used in the initial set of anastomoses. Near-infrared fluorescence imaging technology can assist with flap design and may be useful for assessing the arterial inflow of the flap, but it has not been shown to correlate with flap loss or venous complications. The use of one or two veins in microsurgical free tissue transfer is a topic that has been debated for several years. While some studies indicate that the use of two venous connections may decrease the velocity of blood flow across the anatomosis, there are no data to support differences in flap outcomes or thrombotic events. Therefore, the routine use of a second vein is largely up to surgeon preference. Heparin may be a useful adjunct when thrombosis of the arterial or venous anastomosis is excised and a revision of the anastomosis is performed. Without evidence of thrombosis, it is unlikely to have any added benefit. For venous anastomoses, the use of venous coupling devices has not been associated with patency issues or increased thrombosis rates. Therefore, a hand-sewn anastomosis is not likely to prevent the issue presented in this question.

A 53-year-old woman with a BMI of 27 kg/m2, gigantomastia, and grade III ptosis is considering unilateral mastectomy and autologous reconstruction with abdominal tissue. Which of the following is the most appropriate advice regarding the contralateral breast? A) A contralateral prophylactic mastectomy should be performed to maximize symmetry B) Contralateral reduction should be accompanied by placement of an implant on that side to address upper pole symmetry with the autologous reconstruction C) Simultaneous contralateral reduction may be performed with an acceptable risk profile D) Symmetry is unlikely to be achieved, so a contralateral matching procedure would not be recommended E) Symmetry with autologous tissue is likely to be achieved without the contralateral matching procedure

The correct response is Option C. Studies have shown that contralateral symmetry procedures performed synchronously with unilateral autologous tissue reconstruction after mastectomy (including reconstructions with a free perforator flap) are acceptable. Situations requiring reduction rather than augmentation or mastopexy seem to be the most suited to this timing. A patient with a lower BMI and gigantomastia is unlikely to achieve symmetry without a contralateral procedure. Autologous tissue reconstruction likely would not need a contralateral implant for upper pole symmetry. An attempt at improvement in symmetry using contralateral surgery would be acceptable should the patient so choose, but recommending a prophylactic mastectomy on the contralateral side solely for symmetry and not for risk reduction may be overly aggressive when other methods such as reduction mammaplasty exist with a likelihood of acceptable postoperative symmetry.

A 45-year-old woman comes to the office for consultation regarding severe breast asymmetry after undergoing lumpectomy and radiation therapy for ductal carcinoma of the right breast 5 years ago. Physical examination shows the radiated right breast is tight and retracted, and the left breast is ptotic. Left-sided mastopexy for symmetry and autologous fat grafting to the radiated breast are recommended. The patient asks if the procedure will be covered by insurance. Which of the following is the most appropriate response? A) Fat grafting the right breast will be covered by insurance but the mastopexy will be considered cosmetic and will not be covered B) Insurance companies rarely cover the cost of immediate breast reconstruction C) Insurance may not cover the procedure since insurers are not mandated to pay for reconstruction of lumpectomy defects D) Procedures for both sides will be covered by insurance E) The left-sided mastopexy will likely be covered but the fat grafting will not be covered

The correct response is Option C. The Women's Health and Cancer Rights Act, signed into law in 1998, requires insurance plans to cover the cost of breast reconstruction after mastectomy. The law includes all stages of reconstruction as well as contralateral procedures to provide symmetry. An often misunderstood aspect of the Women's Health and Cancer Rights Act is that it does not apply to individuals undergoing breast conservation therapy (lumpectomy with radiation). As rates of breast conservation therapy have continued to increase (60% of women with stage I cancers), so have significant lumpectomy defects associated with the untoward effects of radiation. In the clinical scenario presented, the patient should be informed that the corrective operation (fat grafting and mastopexy) might not be covered by her insurance company. References Sillah NM, Jee Y, Rao VK, et al. Reconstructive Surgery for Breast Cancer Patients: Have Improved Treatment Modalities Outpaced Current Legislation? Plast Reconstr Surg. 2015 Jul;136(1):123e-124e. Xie, Y, Tang Y, Wehby GL. Federal Health Coverage Mandates and Health Care Utilization: The Case of the Women's Health and Cancer Rights Act and Use of Breast Reconstruction Surgery. J Womens Health (Larchmt). 2015 Aug;24(8):655-62.

A 34-year-old woman is scheduled to undergo bilateral prophylactic mastectomy because of a strong family history of breast cancer. Reconstruction with free flaps from the upper inner thigh is planned. Which of the following is the pedicle to these flaps? A) Ascending branch of the lateral femoral circumflex artery B) Descending branch of the lateral femoral circumflex artery C) Medial femoral circumflex artery D) Superficial circumflex iliac artery E) Superficial femoral artery

The correct response is Option C. The transverse upper gracilis (TUG) flap is a free flap that can be used in breast reconstruction. It is particularly useful in cases where the abdominal donor site is not available or not desired. The flap incorporates the skin and subcutaneous tissues of the upper inner thigh, in the region of the medial thigh lift. The TUG flap is based on the gracilis vessels and perforators from the descending branch of the medial femoral circumflex artery, which forms the dominant pedicle. This arises from the profunda femoris artery. Modifications of this flap have been proposed to increase the volume available for free tissue transfer. The ascending branch of the lateral femoral circumflex artery provides blood supply to the tensor fascia lata flap. The descending branch of the lateral femoral circumflex artery forms the pedicle for the anterolateral thigh flap. The superficial circumflex iliac artery forms the pedicle for the groin flap. Branches from the superficial femoral artery provide minor pedicles to the distal portion of the gracilis muscle, but are less important and distal to the area of the TUG flap.

A 60-year-old woman receives low-molecular-weight heparin (LMWH) 40 U subcutaneously 1 hour before undergoing breast reconstruction using a unilateral transverse rectus abdominis musculocutaneous (TRAM) flap. Weight is 185 lb (84 kg); BMI is 32 kg/m2. Which of the following is the most likely effect of the LMWH on perioperative risks in this patient? A ) Decreased risk of flap failure B ) Decreased risk of postoperative hematoma C ) Decreased risk of pulmonary thromboembolism D ) Increased risk of blood transfusion E ) Increased risk of infection

The correct response is Option C. There are no uniform standards or guidelines for the routine use of chemoprophylaxis of venous thromboembolism in plastic surgery. At a minimum, lower extremity mechanical compression devices should be used on all patients undergoing general anesthesia. The patient described has multiple risk factors for developing deep venous thrombosis (DVT) and pulmonary embolism, such as an age over 50, prolonged surgery time, malignancy, and being overweight. In such high-risk patients, data support the use of chemoprophylaxis with either standard heparin or low-molecular-weight heparin given subcutaneously. As the risk of developing DVT begins with anesthesia induction, it is generally recommended that heparin therapy be started before surgery. Out of concern for bleeding, some surgeons start heparin therapy in the early postoperative period. While more data are needed to clarify the optimal start time of therapy, it is clear that starting therapy before surgical incision is generally safe when dosed appropriately. In large retrospective studies, chemoprophylaxis patients were not more likely to require blood transfusion, though they do demonstrate a slightly greater decrease in postoperative hemoglobin as compared with control patients. There is clearly a decreased risk of postoperative DVT and pulmonary embolism, both clinically apparent and asymptomatic. The data regarding postoperative hematoma are less clear, with some studies showing no increase in €œtake-backs € to the operating room for hematoma with chemoprophylaxis. One study did show an increase in hematomas in oncologic breast surgery with LMWH versus standard, unfractionated heparin. There is no association between flap survival or infection and subcutaneous heparin use. Until reliable prospective, randomized data of sufficient study size are available to demonstrate optimal treatment, the studies seem to support the routine use of mechanical and perioperative subcutaneous heparin prophylaxis in high-risk patients.

A 62-year-old woman with a history of Stage III breast cancer is scheduled for delayed autologous breast reconstruction from the abdominal donor site. She has no other medical problems. BMI is 30 kg/m2. Her mother had a lower extremity deep venous thrombosis in the past. Caprini risk assessment score is 9. Which of the following is the most appropriate method of postoperative VTE risk reduction? A) Aspirin therapy B) Early ambulation after surgery C) Low-molecular-weight heparin therapy D) Sequential compression device use E) No VTE prevention is indicated

The correct response is Option C. Venous thromboembolism (VTE) is a disorder with short-term mortality and long-term morbidity. Plastic and reconstructive surgery patients are known to be at high risk for VTE after surgery. Symptomatic VTE occurs with high frequency after post-bariatric body contouring (7.7%), abdominoplasty (5%), and breast or upper body contouring (2.9%). To fully identify VTE risk in surgical patients, individualized patient assessment is advocated. The Caprini risk assessment model (RAM) is a useful and effective tool to stratify surgical patients for VTE risk. For patients with high Caprini scores, a significantly greater likelihood of VTE events is observed. Approximately 11% of patients with Caprini score >8 will have a VTE within 60 days after surgery. Based upon recommendations from the ASPS VTE Task Force, patients undergoing elective plastic and reconstructive surgical procedures who have Caprini RAM score of 7 or more should have VTE risk reduction strategies employed, such as limiting operating room times, weight reduction, discontinuation of hormone replacement therapy, and early postoperative mobilization. Patients undergoing major plastic and reconstructive operative procedures performed during general anesthesia that last longer than 60 minutes should receive VTE prevention. For patients with Caprini score of 3 to 6, the use of postoperative low-molecular-weight heparin (LMWH) or unfractionated heparin (UH) should be considered. For patients with Caprini score of 3 or more, use of mechanical prophylaxis throughout the duration of chemical prophylaxis for non-ambulatory patients should be considered. For patients with Caprini score of 7 or more, the use of extended LMWH postoperative prophylaxis should be strongly considered. Aspirin does not decrease the risk of VTE and may increase the risk of perioperative complications.

A 42-year-old nurse is scheduled to undergo elective non-implant-based surgery of the left breast. Medical history includes no abnormalities, and she has no allergies. She smokes 1 pack of cigarettes daily. To decrease the incidence of surgical site infection, which of the following is the most effective perioperative strategy? A) Administer cefazolin intravenously within 5 minutes of skin incision B) Administer an insulin drip to keep blood glucose concentration less than 100 mg/dL C) Have the patient practice complete smoking cessation for 7 days prior to surgery D) Prescribe nasal mupirocin and chlorhexidine baths for 5 days prior to surgery E) Use povidone-iodine skin preparation rather than chlorhexidine/isopropyl alcohol

The correct response is Option D. A 2010 randomized controlled trial of over 6700 patients published in the New England Journal of Medicine documented a nearly 60% decrease in Staphylococcus aureus infections in patients if mupirocin was applied twice a day to the nares as well as a full-body wash with chlorhexidine for 5 days prior to surgery. The mean hospital stay was already reduced almost 2 days. A meta-analysis also demonstrated the same findings, with a nearly 45% decrease in surgical site infections (SSIs). Data would suggest that isopropyl alcohol-containing skin preparations for surgery decrease SSI rates more effectively than povidone-iodine alone. Perioperative antibiotics should be administered with enough advance time to achieve proper and adequate rates of skin penetration. With cefazolin, this is 30 to 59 minutes before skin incision, with an odds ratio of 1.0 (vs. 2.0 if given within 30 minutes, and 1.7 if given after 60 minutes). Five minutes prior to skin incision is not sufficient for the SSI-reducing effect to be achieved. Blood glucose control is critical to decreasing SSI rates, with optimal rates usually being quoted as less than 180 mg/dL. However, this patient is not diabetic; she is otherwise healthy, so an insulin drip would not be appropriate. In addition, hypoglycemia can also have detrimental physiologic effects and also should be avoided. Smoking cessation decreases SSI rates if the patient does not smoke for 4 weeks before or after surgery. A 2012 systematic review and meta-analysis of nearly 500,000 patients demonstrated this. However, 7 days of smoking cessation is insufficient time in advance of surgery to obtain these statistically significant benefits. References Bode LG, Kluytmans JA, Wertheim HF, et al. Preventing surgical-site infections in nasal carriers of Staphylococcus aureus. N Engl J Med. 2010 Jan 7;362(1):9-17. Darouiche RO, Wall MJ, Itani KM, et al. Chlorhexidine-alcohol versus povidone-iodine for surgical-site antisepsis. N Engl J Med. 2010 Jan 7;362(1):18-26. Harrison B, Khansa I, Janis JE. Evidence-based strategies to reduce postoperative complications in plastic surgery. Plast Reconstr Surg. 2016 Jan;137(1):351-60. Sørensen LT. Wound healing and infection in surgery: the pathophysiological impact of smoking, smoking cessation, and nicotine replacement therapy: a systematic review. Ann Surg. 2012 Jun;255(6):1069-79. van Rijen MM, Bonten M, Wenzel RP, et al. Intranasal mupirocin for reduction of Staphylococcus aureus infections in surgical patients with nasal carriage: a systematic review. J Antimicrob Chemother. 2008 Feb;61(2):254-61. Weber WP, Marti WR, Zwahlen M, et al. The timing of surgical antimicrobial prophylaxis. Ann Surg. 2008 Jun;247(6):918-26.

A 41-year-old woman comes to the office because of an invasive ductal carcinoma of the left breast. On mammography, the tumor is 3 cm from the nipple and measures 4 cm. A left-sided lateral periareolar scar extending from the 12 o'clock to the 3 o'clock position from a previous biopsy is noted. The patient wishes to undergo a nipple-sparing mastectomy. Which of the following findings places the patient at greatest oncologic risk, including risk for de novo or recurrent cancer or inadequate surgical margins, with this procedure? A) Distance of tumor to nipple B) Patient age C) Presence of the periareolar scar D) Size of tumor E) Type of tumor

The correct response is Option D. As surgical approaches to breast cancer treatment have evolved, nipple-sparing mastectomy (NSM) has emerged as an alternative to other approaches. It was initially used for prophylactic mastectomies, and patients reported increased satisfaction and body image with nipple-areola complex (NAC) preservation. The role of NSM has been expanded to therapeutic mastectomy, and with that there has been increased research in the oncologic safety of this approach. Studies have evaluated therapeutic NSM in the context of invasive ductal carcinoma, invasive lobular carcinoma, and ductal carcinoma in situ. The type of cancer does not appear to be associated with the oncologic safety of NSM. Several studies have demonstrated an inverse association between NAC involvement and distance of the tumor from the nipple. While these studies have varied in their distance cutoffs, nipple involvement is reported to be over 50% when the tumor-nipple distance is less than 2 cm, as noted in one study. There is a direct correlation between tumor size and NAC involvement—the same study cited data that when the tumor was greater than 4 cm, the likelihood of nipple involvement was greater than 50%. One published screening algorithm for plastic surgery includes tumor size less than 3 cm, and tumor location greater than 2 cm from the nipple as criteria for NSM candidacy. A periareolar scar, if large, may compromise the blood supply to the NAC. Acceptable incisions for NSM, however, include a periareolar incision of 25 to 50%.

A 37-year-old woman comes to the office for consultation regarding left breast reconstruction after mastectomy to treat breast cancer. Chemotherapy and radiation therapy are planned postoperatively. She wears a size 34D brassiere. Height is 5 ft 6 in (168 cm), and weight is 160 lb (73 kg). BMI is 25.8 kg/m2. She does not want abdominal scars. Autologous reconstruction with a transverse musculocutaneous gracilis (TMG) flap is planned. Which of the following is a disadvantage of using a TMG flap for this patient's reconstruction? A) Difficulty of flap harvest B) High risk of donor site morbidity C) High risk of fat necrosis D) Small flap size E) Variable vascular anatomy

The correct response is Option D. For breast reconstruction, the TMG flap offers a valuable alternative therapy compared with other standard flaps from the lower abdomen, such as the transverse rectus abdominis musculocutaneous (TRAM) or DIEP flaps. The amount of tissue that can be transferred, however, is limited. The largest flap harvested in one large series weighed 420 g and measured 30 × 10 cm. The flap usually offers enough volume to reconstruct small- to mid-sized breasts. The major advantage of this flap as compared with other flaps, such as the gluteal flap or the perforator latissimus flap, is its constant vascular anatomy. Flap perfusion is always reliable, with low rates of fat necrosis and tissue similar in consistency with breast tissue. The donor scar is inconspicuous. The incision is comparable to incisions for a thigh lift and is well hidden. A distortion of the labia majora is typically not observed. However, as in thigh lifts, a lowering of the scar remains a possible problem. Functional donor-site morbidity after TMG flap harvest itself is low. After having clinically established the transverse approach for routine procedures, harvesting one flap is rapid and usually takes no longer than 30 minutes.

A 40-year-old woman is referred for reconstruction following mastectomy for a peripherally located ductal carcinoma in situ. A nipple-sparing mastectomy with immediate, single-stage prosthesis reconstruction with acellular dermal matrix is planned. Which of the following interventions is most appropriate to ensure that no residual cancer exists? A) Chemotherapy B) Ductal washing C) Radiation therapy D) Retroareolar frozen section E) Sentinel lymph node evaluation

The correct response is Option D. More plastic surgeons are performing reconstruction for women pursuing prophylactic mastectomy, which is requested quite frequently to avoid cancer recurrence and to achieve optimal aesthetic outcome. Exclusion criteria for nipple-sparing mastectomy include tumors greater than 3 cm, clinical invasion of the nipple-areola complex, tumors within 2 cm of the nipple, evidence of multicentric disease, positive intraoperative retroareolar frozen section, or nodal disease. If carcinoma is found in the retroareolar tissue, the nipple-areola complex must be removed. A patient who would require sentinel lymph node evaluation, radiation therapy, or chemotherapy would not be an appropriate candidate for nipple-sparing mastectomy. Annual mammography is recommended for any patient with a history of breast cancer and is not specific to the issue of nipple-sparing mastectomy. Ductal washing is not relevant for this pathology.

Which of the following characteristics is correlated with increased risk of nipple-areola complex necrosis in nipple-sparing mastectomies with immediate reconstruction? A) Autologous tissue reconstruction B) Direct to implant reconstruction C) Patient age D) Periareolar incision E) Small breast size

The correct response is Option D. Nipple-sparing mastectomies (NSMs) are becoming more common for both therapeutic and prophylactic mastectomies. Nipple-areola complex (NAC) necrosis can imperil reconstructive efforts, as well as negatively affect patients emotionally. It is important to maximize perfusion to the mastectomy skin flaps and NAC while still performing an oncologically sound procedure. There are multiple different incisions for performing NSM. Periareolar, inframammary-fold, radial, and vertical incisions are the most common. Periareolar incisions are associated with an increased risk of NAC necrosis in NSMs. Type of reconstruction, small breast size, and patient age have not been shown to be linked to increased rates of NAC necrosis.

A 43-year-old Caucasian woman is referred to the office because of a mass on her right breast that has been rapidly growing for 8 weeks. Physical examination shows a 4.5-cm, freely movable mass in the right breast. No axillary adenopathy or nipple discharge is noted. Which of the following is the most likely nonepithelial neoplasm in this patient? A) Fibroadenoma B) Hamartoma C) Lipoma D) Phyllodes tumor E) Primary breast lymphoma

The correct response is Option D. Primary breast lymphoma is rare; it constitutes less than 0.6% of all breast malignancies. Phyllodes tumors represent about 1% of tumors in the breast; they are the most commonly occurring nonepithelial neoplasm of the breast. The tumor has a smooth, sharply demarcated texture and is generally freely movable. It is a relatively large tumor, with an average size of 5 cm. A hamartoma is the most common benign tumor of the lung. Hamartomas of the breast are benign tumors composed primarily of dense fibrous tissues with variable amounts of fat and associated ducts. Eighty-five percent of phyllodes tumors are benign, and 15% are malignant. There is no race predilection, but phyllodes tumors occur almost exclusively in women. In young women under age 25 years, asymmetric, tender, and fibrocystic tissues usually point to a fibroadenoma or circumscribed fibrocystic mass. Lipoma of the breast causes diagnostic and therapeutic uncertainty. Clinically, it may be difficult to distinguish a lipoma from other conditions. Fine-needle aspiration cytology is often not helpful. Both mammography and ultrasonography results are often negative. MRI may be overread, but it is useful as a diagnostic tool. This neoplasm can be seen in men and women.

A 42-year-old woman with a 3-cm invasive ductal carcinoma of the right breast is evaluated for breast reconstruction. She has not decided how she wants to manage her contralateral breast. Regarding eliciting a family history, which of the following cancers is associated with a mutation in a breast cancer-susceptibility gene? A) Colon B) Esophageal C) Lung D) Pancreatic E) Thyroid

The correct response is Option D. The breast cancer-susceptibility gene types 1 and 2 (BRCA1 and BRCA2) are tumor suppressor genes. Mutations in BRCA1 and BRCA2 are associated with hereditary breast and ovarian cancers. Additionally, they can be associated with increased risks of pancreatic and prostate cancer. Thyroid, lung, esophageal, and colon cancer are not associated with increased risks of BRCA1 and BRCA2 mutations.

A 35-year-old woman comes to the office for consultation regarding prophylactic mastectomy and breast reconstruction. The patient's mother and sister were diagnosed with bilateral breast cancer in their premenopausal years. Genetic testing for BRCA mutations is negative. Which of the following best estimates this patient's lifetime risk of breast cancer? A) 5% B) 13% C) 20% D) 45% E) 80%

The correct response is Option D. The cumulative lifetime risk for a 35-year-old woman whose mother and sister had breast cancer is estimated to be approximately 15%. The risk may increase to as high as 45% if those cancers were premenopausal and bilateral. BRCA hereditary cancer is characterized by autosomal dominant genetics with multiple family members in each generation being affected. For patients with BRCA1 mutation, the risk of breast cancer has been estimated to be between 50 and 80% by age 65 years. The risk of developing ovarian cancer has been estimated to be 10% by age 60 years.

From 2015, lady with huge reconned breasts A 55-year-old woman comes to the office for a second opinion because she is displeased with the results of a recent bilateral mastectomy and breast reconstruction with 800-mL high-profile silicone implants. A photograph is shown. BMI is 35 kg/m2. She repeatedly shows pictures of models with augmented breasts and says that she wants her breasts to be "perkier." She requests augmentation/mastopexy. Which of the following is the most appropriate next step in management? A) Augmentation/mastopexy B) Implant exchange C) Mastopexy D) Reassurance E) Referral to a psychiatrist

The correct response is Option D. The most reasonable approach in this patient is to offer reassurance and reset her expectations. A patient with a BMI of 35 kg/m2 who undergoes mastectomy and implant reconstruction will never look like a model with augmented breasts. This patient clearly has misguided expectations. Any surgical intervention is unlikely to produce the result she is looking for, when in fact she has a very acceptable result as is. Referral of this patient to a psychiatrist will likely upset the patient and undermine her trust.

2017 and below Following a skin-sparing mastectomy, a 39-year-old woman undergoes deep inferior epigastric perforator (DIEP) flap breast reconstruction. To augment flap sensation, the anterior sensory branch of the fourth intercostal nerve is coapted to which of the following nerves within the DIEP flap? A) Genitofemoral B) Iliohypogastric C) Ilioinguinal D) Intercostal E) Lateral femoral cutaneous

The correct response is Option D. The third, fourth, and fifth intercostal nerves are responsible for innervation of the majority of the breast. The anterior branch of the fourth intercostal nerve provides most erogenous sensation to the nipple. Sensation to the lower abdomen arises from segmental cutaneous branches of the intercostal nerve, which travel through the rectus abdominis muscle. T10 provides sensation to the dermatome, including the periumbilical region, and is most commonly used. The iliohypogastric nerve provides sensation to the lateral gluteal region. The ilioinguinal nerve provides sensation to the upper medial thigh. The genitofemoral nerve provides sensation to the upper anterior thigh and mons pubis. The lateral femoral cutaneous nerve provides innervation to the lateral thigh and is not used for this purpose. References Temple CL, Ross DC, Kim S, et al. Sensibility following innervated free TRAM flap for breast reconstruction: Part II. Innervation improves patient-rated quality of life. Plast Reconstr Surg. 2009 Nov;124(5):1419-25. Temple CL, Tse R, Bettger-Hahn M, MacDermid J, Gan BS, Ross DC. Sensibility following innervated free TRAM flap for breast reconstruction. Plast Reconstr Surg. 2006 Jun;117(7):2119-27.

An otherwise healthy 67-year-old woman with advanced breast cancer is scheduled to undergo mastectomy and immediate reconstruction with a free transverse rectus abdominis musculocutaneous (TRAM) flap. BMI is 35 kg/m2. Which of the following is most appropriate for deep venous thrombosis prophylaxis? A) Aspirin and intermittent pneumatic compression stockings B) Elastic compression stockings only C) Intermittent pneumatic compression stockings only D) Low-molecular-weight heparin and intermittent pneumatic compression stockings E) Positioning and early ambulation

The correct response is Option D. This patient is at highest risk for deep venous thrombosis and she will require combination therapy of compression stockings and chemical prophylaxis. Perhaps the most well-regarded set of guidelines in this matter comes from the American College of Chest Physicians. These guidelines provide treatment recommendations based on a patient's risk classification. Caprini model offers a very user-friendly method of calculating patient risk factors and categorizing them into low, moderate, high, and highest risks. The patient in question has risk factors (age, malignancy, and major surgery) that would put her in the highest risk category. In the highest risk category, prophylaxis will require combination therapy or warfarin.

A 46-year-old woman with ductal carcinoma in situ is scheduled to undergo right mastectomy. Immediate reconstruction with a tissue expander and acellular dermal matrix (ADM) is planned. Which of the following is an expected outcome with use of ADM? A) Decreased formation of seroma B) Decreased incidence of hematoma C) Decreased risk of infection D) Increased capsule contracture E) Increased initial fill of the expander

The correct response is Option E. According to Sbitany, et al, acellular dermal matrix (ADM) allows for a greater initial fill of saline. This potentially improves cosmetic outcome, as it better capitalizes on preserved mastectomy skin for reconstruction. Sbitany, et al, concluded that ADM-assisted prosthesis breast reconstruction has a safety profile no worse than that of complete submuscular coverage but offers the benefit of fewer expansions and the potential for more predictable secondary revisions. ADM has enhanced prosthesis-based reconstruction and remains useful in immediate prosthetic breast reconstruction. However, it has been found to have higher rates of postoperative seroma and infection. It has also been reported to decrease capsule contracture.

Which of the following is the most accurate location of the elliptical skin island of a profunda artery perforator (PAP) flap? A) Anteromedial thigh with the superior border within the groin crease B) Inferior buttock with the inferior border within the gluteal fold C) Lateral hip superior to the iliac crest D) Middle buttock, from the posterior superior iliac spine to the apex of the greater trochanter E) Posteromedial thigh with the superior border within the gluteal fold

The correct response is Option E. According to the literature, the skin island of the profunda artery perforator (PAP) flap is inferior to the gluteal fold.1,2 An ellipse of the inferior buttock with the inferior border within the gluteal fold describes the skin island of the inferior gluteal artery perforator (IGAP) free flap.3 An ellipse of the anteromedial thigh with the superior border within the gluteal fold describes the transverse upper gracilis (TUG) flap. An ellipse of the middle buttock, from the posterior superior iliac spine to the apex of the greater trochanter, describes the superior gluteal artery perforator (SGAP) flap.3 An ellipse of the lateral hip superior to the iliac crest describes the Rubens or lateral hip flap.4 The only option that correctly identifies the skin island for the PAP flap is an ellipse of the posteromedial thigh with the superior border within the gluteal fold. The superior marking is within or just below the gluteal fold and the inferior marking is roughly 7 cm below the superior marking. The flap is an ellipse so the scar does not extend outside of the gluteal fold.1,2

A 53-year-old woman comes to the office because of ulcerated tissue 6 weeks after undergoing radiation therapy for breast cancer. She underwent mastectomy 1 year ago. Analysis of the radiated tissue is most likely to show an increase in which of the following? A ) Acute inflammatory response B ) Cytokines and growth factors C ) Neutrophil function D ) Tissue oxygenation E ) Vessel thrombosis

The correct response is Option E. Analysis of radiated tissue will show increased vessel thrombosis. Ionizing radiation directly damages the genome or injures the DNA through free radical production. In acute radiation injury, the skin becomes erythematous and edematous with the dilation of fine blood vessels, endothelial edema, and lymphatic obliteration. Although perfusion of the skin assessed with fluorescein injection appears normal, tissue oxygenation is inadequate. Healing is impaired with slowed fibroblast proliferation and impairment of the acute inflammatory response. Fibroblast defects are the main problem in the inhibited healing of chronic radiation injury. Phagocytosis and bacteriocidal metabolic functions in neutrophils are also impaired. This effect increases after therapy, which cannot be a direct effect of radiation on the neutrophils because their lifespan is too short. The local wound environment and irradiated tissue do not prime the neutrophils with the appropriate cytokines and growth factors needed for activation. This may lead to an increased incidence of postoperative infections in patients with previous radiation. Recent studies have shown promise in the treatment of radiation tissue damage with lipoaspirate transplantation. Adipose-derived stem cells have been hypothesized to target damaged areas, release angiogenic factors, form new vessels, and increase tissue oxygenation.

An otherwise healthy 37-year-old woman presents for delayed microsurgical breast reconstruction. Which of the following is associated with use of tamoxifen? A) Hemodynamic instability B) Impaired wound healing C) Increased bleeding D) Seroma formation E) Thromboembolic events

The correct response is Option E. Breast cancers that are estrogen receptor positive may be responsive to adjuvant chemotherapy with selective estrogen receptor modulators such as tamoxifen, which can reduce recurrence and mortality. Tamoxifen is associated with thromboembolic events, such as deep venous thrombosis and pulmonary embolism. This prothrombotic effect has been postulated to be secondary to the effect of tamoxifen on estrogen receptors that are abundant within vascular endothelium. Tamoxifen has been shown to be associated with increased rates of total flap loss and decreased rates of flap salvage when taken within 28 days of microsurgical breast reconstruction, which represents two half-lives of the active metabolite of tamoxifen (N-desmethyl tamoxifen, t1/2=14 days). It has therefore been recommended that in patients undergoing microsurgical breast reconstruction, tamoxifen be held for at least 28 days preoperatively. Some authors have further advised holding the medication postoperatively in addition to preoperatively. Tamoxifen is not associated with impaired wound healing, increased bleeding, hemodynamic instability, or seroma formation.

A 43-year-old woman would like to discuss plans for breast reconstruction after her upcoming unilateral mastectomy. Postoperative radiation therapy is planned. Which of the following is the most likely benefit of tissue expander-based breast reconstruction compared with immediate autologous breast reconstruction using this patient's abdominal tissue? A) Better symmetry B) Improved postoperative sensation C) A larger, more ptotic breast reconstruction D) Lower risk of complications from radiation E) Preservation of the patient's options for final reconstruction

The correct response is Option E. Immediate breast reconstruction with tissue expanders followed by reconstruction of choice preserves the patient's skin envelope and keeps open options for definitive final reconstruction of choice whether with an implant or autologous tissue. Tissue expander-based reconstruction is associated with a higher complication rate in the setting of radiation therapy but preserves abdominal and back tissue as options for autologous reconstruction. Implant-based reconstruction does not provide the advantages of improved symmetry, sensation, or breast ptosis.

A 42-year-old woman who underwent mastectomy of the right breast 6 months ago is evaluated for delayed autologous breast reconstruction with free tissue transfer. Which of the following medications should be discontinued preoperatively if she is routinely taking it? A) Diltiazem B) Fluoxetine C) Metoprolol D) Multivitamin E) Tamoxifen

The correct response is Option E. It is well known that tamoxifen can increase the risk of thrombembolic events. In a retrospective study at MD Anderson Cancer Center, it was shown that patients who received the drug close to the procedure had a significantly higher rate of complications. It is recommended that the patient stop tamoxifen at least 28 days before surgery. Other listed medications do not have a direct effect on thrombotic complications.

A 30-year-old woman comes to the office because of pain in the left breast. Two weeks ago, she underwent core needle biopsy of a breast mass that was diagnosed as benign. Family history does not include breast cancer. She does not smoke cigarettes. On examination, the left breast is erythematous and tender to palpation, and the skin of the breast is retracted laterally. There is a palpable, rope-like mass that courses longitudinally along the breast. Which of the following is the most appropriate diagnosis and treatment of this lesion? A) Breast abscess, perform incision and drainage of the mass B) Fibrocystic changes, perform biopsy C) Mastodynia, treat with 10-day course of broad-spectrum antibiotics D) Nipple papilloma, perform diagnostic mammography E) Superficial thrombophlebitis, manage with analgesics

The correct response is Option E. Mondor disease, or superficial thrombophlebitis of the breast, involves the superficial veins of the breast and anterior chest wall. It may occur following surgery, core biopsy, irradiation, or trauma. Clinical manifestations include pain, redness and swelling, and the presence of a thickened tender cord. This condition usually resolves in 4 to 6 weeks with symptomatic treatment using pain relief. Nipple papillomas may be identified as a mass on breast imaging or may be found incidentally. They frequently present with bloody nipple discharge. While not concerning in and of themselves, these lesions may harbor areas of atypia or ductal carcinoma in situ, and are treated with core needle biopsy. While a breast abscess is possible after a diagnostic procedure such as a biopsy, it would present as localized swelling, tenderness, and induration. The skin retraction and rope-like mass would not be present. Fibrocystic changes in the breast present as a solitary mass and may cause patients to seek medical attention because of associated pain. They may fluctuate in size and tenderness during a patient's menstrual cycle. Because no breast mass can be definitively declared benign on physical examination alone, imaging and/or biopsy may be considered. Breast pain in the absence of a finding on physical examination may have a number of causes including menstrual changes, breast hypertrophy, diet, hormone replacement therapy, ductal ectasia, mastitis, malignancy, and hidradenitis. While the patient does have breast pain, the other physical findings rule out mastodynia alone. References Courtillot C, Plu-Bureau G, Binart N, et al. Benign breast diseases. J Mammary Gland Biol Neoplasia. 2005 Oct;10(4):325-35. Laroche JP, Galanaud J, Labau D, Van Kien AK, Brisot D, Boge G, Quéré I. Mondor's disease: what's new since 1939? Thromb Res. 2012 Oct;130 Suppl 1:S56-8. Salemis NS, Vasilara G, Lagoudianakis E. Mondor's disease of the breast as a complication of ultrasound-guided core needle biopsy: management and review of the literature. Breast Dis. 2015;35(1):73-6.

Nipple-sparing mastectomy is CONTRAINDICATED in which of the following women considering mastectomy and immediate breast reconstruction? A) A 21-year-old with BRCA1 mutation and grade I breast ptosis B) A 31-year-old with BRCA2 mutation; subglandular augmentation mammaplasty 1 year ago C) A 45-year-old with unilateral breast cancer and need for postoperative radiation therapy D) A 50-year-old with unilateral breast cancer in the tail of Spence E) A 61-year-old with unilateral breast cancer; tumor-to-nipple distance of 1 cm

The correct response is Option E. Nipple-sparing mastectomy is increasingly prevalent owing to perceived improvement in reconstructive outcome and patient satisfaction. Prevalence of nipple ischemia, which can vary in severity from incomplete, partial-thickness epidermolysis to total nipple loss, ranges from 2 to 60% depending on the definition used. A recent pooled analysis found a cumulative prevalence of 7%. Most cases respond to local wound care; reoperation for total nipple loss is relatively infrequent. Patient selection remains paramount to successful nipple preservation. Typically, thin, non-smoking patients with small, non-ptotic breasts are considered ideal candidates. Severe macromastia and ptosis may not only increase risk of poor nipple vascularity, but also contribute to nipple malposition. Oncologic determinants of the safety of nipple preservation should also be considered. Noninflammatory cancers located in excess of 2 cm from the nipple can generally be safely extirpated without removal of the nipple. Usually, an intraoperative frozen section biopsy of the retroareolar tissue ("doughnut") is performed after excision of the main mastectomy specimen to demonstrate nipple margins free of tumor. Patients undergoing prophylactic mastectomy in the setting of high genetic predisposition are also considered good candidates. Successful nipple preservation has also been described in patients with a history of reduction mammaplasty or mastopexy. Successful nipple-sparing mastectomy with implant reconstruction has been described recently in patients who require postoperative radiation, albeit with higher risk of capsular contracture and nipple malposition. Prior augmentation in the subglandular or subpectoral positions does not contraindicate nipple-sparing mastectomy; rather, the latter group may present opportunities for direct-to-implant reconstruction. Location of a breast cancer in the tail of Spence (axillary extension of breast tissue) should not contraindicate nipple-sparing mastectomy, as it is likely to be far enough from the nipple to safely spare it.

Which of the following is a risk factor for hormone-sensitive breast cancer? A) Breast-feeding B) Early age at first pregnancy C) Early menopause D) Late menarche E) Post-menopausal obesity

The correct response is Option E. Post-menopausal obesity is associated with increased adipose production of estrogen, which can increase the risk for hormone-sensitive breast cancer. Other options (late menarche, early menopause, and breast feeding) decrease the number of menstrual cycles, and therefore may decrease the risk of breast cancer. Early age at first pregnancy is also associated with decreased risk for hormone-sensitive breast cancers. References Mathes SJ, Lange J. Breast Cancer: Diagnosis, Therapy, and Postmastectomy Reconstruction. Mathes SJ, ed. In: Plastic Surgery. Philadelphia, PA: Elsevier, Inc; 2006:631-2. What are the Risk Factors for Breast Cancer? American Cancer Society. Available at: http://www.cancer.org/cancer/breastcancer/detailedguide/breast-cancer-risk-factors. Published December, 2015.

A 45-year-old woman comes to the office for consultation regarding mastectomy and immediate breast reconstruction because of recurrent right-sided breast cancer. She underwent breast-conserving therapy and radiation therapy 5 years ago. BMI is 23 kg/m2. The patient is otherwise healthy, and she does not smoke cigarettes. On examination, the breast is soft with obvious fibrosis. Use of which of the following has the highest risk for reconstructive failure in this patient? A) Deep inferior epigastric artery perforator (DIEP) flap B) Free transverse rectus abdominis musculocutaneous (TRAM) flap C) Latissimus flap with prosthesis D) Pedicled TRAM flap E) Tissue expander and prosthesis

The correct response is Option E. Pre-reconstruction radiation is typically seen in two groups of patients: those who underwent mastectomy without reconstruction followed by radiation therapy, or those who underwent breast-conserving therapy and radiation with recurrence or new cancer. Although these patients may present with a reasonable skin envelope, complication rates associated with tissue expander/prosthesis have been reported as high as 70%, with a 40% rate of failure or conversion to flap. Additionally, aesthetic outcomes in most patients who completed expander/prosthesis reconstruction after previous radiation therapy were deemed good or very good compared with the majority of non-irradiated patients who achieved excellent results. Tissue expander/prosthesis can be considered as an option for patients with a history of previous irradiation who wish to avoid the scars and recovery of flap-type reconstructions; however, they should be counseled of the high risks of complications

A 45-year-old woman who has breast cancer comes to the office for consultation regarding bilateral breast reconstruction. Reconstruction using autologous abdominal tissue is considered. The risk of abdominal morbidity is discussed. Which of the following flap techniques is most likely to result in the lowest level of overall abdominal morbidity? A ) Deep inferior epigastric artery perforator B ) Free muscle-sparing transverse rectus abdominis musculocutaneous (TRAM) C ) Free TRAM D ) Pedicled TRAM E ) Superficial inferior epigastric artery flap

The correct response is Option E. The superficial inferior epigastric artery (SIEA) flap results in the lowest level of overall abdominal morbidity, as the technique used in harvesting this flap leaves the abdominal fascia intact. These vessels are only present in less than one third of patients, and only one half of those patients will have vessels of sufficient diameter to support a free tissue transfer. The SIEA flap is also associated with a higher frequency of total flap loss, in addition to a higher incidence of fatty necrosis. The deep inferior epigastric artery perforator (DIEAP or DIEP) flap involves dissection of one or two (occasionally more) perforators through the rectus muscle to the inferior epigastric vessels. Although this technique does not include any rectus muscle or sheet/fasica within the flap itself, it does involve moderate-level trauma to those organs and can cause abdominal wall morbidity. The free TRAM or free muscle-sparing TRAM techniques are free tissue transfer variants of the TRAM, whereby a small amount of the rectus is taken with the flap. In the more advanced MS free TRAM technique, the amount of muscle taken is only enough to allow safe transfer of the perforators. Although this technique is less invasive than the pedicled TRAM, it does still involve removal of a variable portion of the rectus muscle and fascia. A pedicled TRAM flap technique involves transferring of the flap based on the superficial epigastric vessel that runs within the rectus muscle. Therefore, the entirety of the rectus (unilateral or bilateral) is elevated out of its native abdominal wall location. Many publications have compared the other modes of breast reconstruction with reference to abdominal wall morbidity. This area remains controversial. The general consensus remains that in a bilateral reconstruction, pedicled TRAM flaps are associated with higher levels of overall abdominal morbidity (hernias, bulges, weakness, intolerance to exercise, etc) when compared with the use of MS free TRAM, DIEP, or SIEA flaps. The use of SIEA flaps results in minimal to no abdominal wall morbidity.

A 60-year-old woman with breast cancer undergoes a transverse rectus abdominis musculocutaneous flap breast reconstruction after mastectomy. She has no allergies. Weight is 200 lb (91 kg). Estimated blood loss is 200 mL. Duration of the operation is 3 hours and 50 minutes. Administration of cefazolin before skin incision is planned as prophylaxis against surgical site infection. Which of the following is the most appropriate dosage and timing of this injection? Dose Timing Redosing A) 1g 5 minutes prior after 150 mL of blood loss B) 1g 15 minutes prior no C) 1g 40 minutes prior no D) 2g 15 minutes prior no E) 2g 40 minutes prior no

The correct response is Option E. There has been a renewed interest in perioperative antibiotics in recent years toward more appropriate use to decrease surgical site infection (SSI) while decreasing the incidence of resistant bacteria. Current recommendations are to administer a single perioperative dose of antibiotics against common skin flora (gram positive), usually using a first-generation cephalosporin. However, the following recommendations may be underappreciated: Cefazolin intravenous: 1 g if <80kg; 2g if >80kg Alternatives: clindamycin 600 to 900mg intravenously; vancomycin 1 to 1.5g intravenously In this case, the patient weighs 200lb (91kg), so 2g of cefazolin is the recommended dosage. Additionally, an important factor is the timing of the administration of the perioperative antibiotics in order to achieve proper skin levels before incision. In one study by Classen et al., published in the New England Journal of Medicine in 1992, the optimal time was between 2 hours before the operation and skin incision, as greater than 2 hours before and any time after skin incision led to marked increases in the relative risk of SSIs. A follow-up by Weber et al. in 2008 narrowed the most appropriate window to between 30 and 59 minutes before skin incision. Finally, there is the issue of redosing. Current recommendations are to redose if there is excessive blood loss (>1500 mL) or if there are long procedures where one exceeds the half-life of the antibiotic used. In the clinical scenario described, the most appropriate choice is 2 g of cefazolin, because the patient's weight is above 80 kg, administered between 30 to 59 minutes before skin incision. Redosing on the basis of blood loss is unnecessary, although one could consider redosing at approximately 4 hours on the basis of half-life.

This is from 2013 left chest wound with exposed lung and heart A 48-year-old woman undergoes radical resection of the left breast, left hemisternectomy, four-rib resection, and visceral pleurectomy because of invasive inflammatory breast cancer extending through the thoracic wall into the mediastinum and chest. Examination of specimens obtained from intraoperative frozen section biopsies shows no residual disease. Postoperative adjuvant chemotherapy and radiation therapy to the area are planned. An intraoperative photograph of the 17 × 17-cm defect is shown. Which of the following is the most appropriate technique for soft-tissue reconstruction? A) Left latissimus muscle flap with skin graft B) Left rectus muscle turnover flap and skin graft C) Omental flap and skin graft D) Reverse abdominoplasty flap E) Right transverse rectus abdominis musculocutaneous (TRAM) flap

The correct response is Option E. The most appropriate option for this patient would be a right TRAM flap. The defect encompasses a composite defect of the left side of the sternum, including the left internal mammary artery, four contiguous ribs, and the entire left breast. The question specifically asks for soft-tissue thoracic reconstruction. The analysis of the defect yields a very large soft-tissue requirement. Given this, the most logical choice of flaps to provide this amount of soft tissue on a reliable pedicle would be a right (contralateral) TRAM flap. Furthermore, this flap could also be shaped to provide a breast mound versus all the other choices. A left latissimus muscle flap with skin graft is not the optimal choice as it would not provide sufficient soft-tissue coverage with its associated skin paddle without undue donor site morbidity in this particular patient with a large defect that spans parasternal and lateral chest wall. As a muscle-only flap with skin graft, however, it may be considered as a backup option in case of primary flap failure. An omental flap with skin graft is an option for reconstruction of this defect; however, it would not be the most appropriate option in this case as it would necessitate an otherwise unnecessary intra-abdominal procedure and would not reshape a breast mound. It has been shown, however, to be optimal in contaminated areas due to its associated lymphatic vessels and nodes associated with its pedicle, so in different circumstances, this may be a preferred choice. A left rectus turnover flap with skin graft would not be a good option for two reasons: the left internal mammary artery has been harvested, thereby compromising the superior epigastric vessel on which this flap would be based; and it would not cover the entirety of the defect. A reverse abdominoplasty flap would not be able to reach the full superior extent of the defect.


Conjuntos de estudio relacionados

Chapter 2 - Final Exam - MIE 330

View Set

intro to Communication: Exam Q&As

View Set

Environmental Science Chapter 10

View Set

Ch. 25: Emergency Management and Preparedness

View Set